You are on page 1of 3

Math 25, Fall 2014.

HW 1 Solutions (mostly adapted from Abbotts Instructors Manual)

1.2.1. (a) We prove this by contradiction. Assume that there exist integers p and q satisfying
 2
p
= 3.
q

We may assume that p and q have no common factor. From the above equation we get

p2 = 3q 2 .

Therefore 3|p2 and hence 3|p. This allows us to write p = 3k where k is an integer. After substituting
3k for p, we get (3k)2 = 3q 2 , which can be simplified to 3k 2 = q 2 . This implies 3|q 2 , then 3|q. Thus
we have shown p and q have a common factor, namely 3. But they were originally assumed to have
no common factor, a contradiction.
A similar argument will work for 6 as well because we get p2 = 6q 2 which implies p is a multiple
of both 2 and 3, hence a multiple of 6. After the argument as above, we conclude q is also a multiple
of 6. Therefore, 6 must be irrational.
(b) In this case, the fact that 4|p2 does not imply 4|p. Thus, the above proof breaks down at this
point.

1.2.2. (a) False, as seen in Example 1.2.2.


(c) False. Consider sets A = {1, 2, 3}, B = {3, 6, 7}, and C = {5}. Note that A (B C) = {3} is not
equal to (A B) C = {3, 5}.
(d) and (e) are both true.

1.2.5. (a) Observe that |a b| = |a + (b)| |a| + | b| = |a| + |b| which implies |a b| |a| + |b|.
(b) First note that |a| = |a b + b| |a b| + |b|. Taking |b| to the left side of the inequality we get
|a| |b| |a b|. Reversing the roles of a and b in the previous argument gives |b| |a| |b a|, thus
(after multiplying by 1), |b a| |a| |b| Because |a b| = |b a|, the result follows.

1.2.6. (c) We have to show that y g(A B) implies y g(A) g(B). If y g(A B), then there
exists an x A B with g(x) = y. But this implies that x A and x B and hence g(x) g(A)
and g(x) g(B). Therefore, g(x) = y g(A) g(B).

1.2.7. (b) We verify this by definition:

x g 1 (A) g 1 (B)
x g 1 (A) and x g 1 (B)
g(x) A and g(x) B
g(x) A B
x g 1 (A B).

1
Similarly,
x g 1 (A) g 1 (B)
x g 1 (A) or x g 1 (B)
g(x) A or g(x) B
g(x) A B
x g 1 (A B).

1.2.8. (a) There exist two real numbers a and b satisfying a < b, such that for all n N we have
a + 1/n b.
(b) There exist two distinct rational numbers with the property that every number in between them
is rational.

(c) There exists a natural number n, such that n is a rational number but not a natural number.
(d) There exists a real number x, such that, for all n N, n x. (We often express the same
statement in a different word order to save commas: There exists a real number x, such that n x
for all n N).

1.2.10. (a) For n = 1, we check: y1 = 1 < 4. For the induction step, we want to show that if we
have yn < 4, then it follows that yn+1 < 4. Starting from the induction hypothesis yn < 4, we get
yn+1 = 43 yn + 1 < 43 4 + 1 = 4, which is the the desired conclusion yn+1 4. By induction, the claim
is proved for all n N.
(b) For n = 1, we check y1 = 1 < 7/4 = y2 , proving the base case. For the induction step, we want
to show that if we have yn yn+1 , then it follows that yn+1 yn+2 . Starting from the induction
hypothesis yn yn+1 , we can multiply across the inequality by 3/4 and add 1 to get 34 yn +1 43 yn+1 +1
which is the the desired conclusion yn+1 yn+2 . By induction, the claim is proved for all n N.

1.2.12. (a) The check for n = 2 is easy and is omitted. For the induction step, we want to show that
if we have
(A1 . . . An )c = Ac1 . . . Acn ,
then it follows that
(A1 . . . An+1 )c = Ac1 . . . Acn+1 .
But we have
(A1 . . . An+1 )c = ((A1 . . . An ) An+1 )c
which by n = 2 case equals
(Ac1 . . . Acn ) Acn+1
and we drop the parentheses to get the desired conclusion. By induction, the claim is proved for all
n N.
(b) The point here is to distinguish between asserting that a statement is true for all values of n N
and asserting that it is true in the infinite case. Induction cannot be used when we have an infinite
number of sets: it is used to prove facts that hold true for each value of n N, bur then we cannot
just plug in n = !
(c) By definition x 6
n=1 An exactly when the negation of the following statement is true:

(n N)(x An ).

But the negation is equivalent to

(n N)(x 6 An ) (n N)(x Acn ) x c


n=1 An .

2
1.3.3 (a). The point of this problem is that existence of the greatest lower bound follows form the
Axiom of Completeness, that is, from the existence of the least upper bound.
Because A is bounded below, B 6= . The first observation is that, for all a A and b B, we
have b a. This tells us is that B is bounded above and thus = sup B exists by the Axiom of
Completeness.
Fix an a A. We have observed that a is an upper bound for B; we conclude that = sup B a.
As this is true for every a A, is a lower bound for A. Thus B, and so = max B. This last
fact states exactly that is the largest of all lower bounds for A, i.e., that = inf A. In particular,
inf A exists.
Remark . Another way to solve this problem is to show first that inf A exists. One way to do
this is by considering the set A = {a : a A}. Then we need a few verifications (which are
good exercises). First we check that, as A is bounded below, A is bounded above. Thus sup(A)
exists. Then we check that sup(A) is the greatest lower bound for A, that is, that inf A exists and
inf A = sup(A). Once we know that inf A exists, Problem 1.3.3 (a) becomes easy: by definition of
greatest lower bound, inf A = max B, and then since max B exists it must equal to sup B.

1.3.4. Observe that A contains all elements of B and hence sup A b for all b B. By part (ii) of the
definition of supremum, sup B is less than or equal to any other upper bound of B. Because sup A is
an upper bound for B, it follows that sup B sup A.

1.3.6. (a) The supremum is 3 and the infimum is 1.


(b) The supremum is 1 and the infimum is 0.
(c) The supremum is 1/2 and the infimum is 1/3.
(d) The supremum is 9 and the infimum is 1/9.

1.3.7. Since a is an upper bound for A, we only need to verify the second part of the definition of
supremum and show that if d is any upper bound for A then a d. However, by the definition of
upper bound, x d for every x A, in particular for x = a.

1.3.9. (a) True. One of the numbers in the set is the largest, and that number is the supremum.
(c) False. Consider A = (0, 1) and B = (1, 2). Then sup A = 1 = inf B.

1.4.2. (a) omitted. For (b), argue by contradiction: if a + t is rational, then t = (a + t) a is also
rational (and similarly
for the product). For (c), the answer is no for both addition and multiplication.
Consider a = 2, b = a. Then a and b are irrational, but a + b = 0 and ab = 2 are both rational.

1.4.5. Denote A = n=1 (0, 1/n). Take an arbitrary x R. We need to show that x / A. This is clear
if x 0 as then x
/ (0, 1/n) for every n N. If x > 0, then we know there exists an n0 N such that
1/n0 < x, and so x / (0, 1/n0 ). This implies x
/ A and our proof is complete.

1.4.6. Omitted as the proof is practically identical to the one for 2.

You might also like